0% found this document useful (0 votes)
277 views56 pages

Laplace_Transform

The document is an educational resource on Laplace Transforms, authored by Jeeja A. V. for a mathematics course at the University of Kerala. It covers various topics including the definition, existence, linearity, and transforms of elementary functions, along with proofs and examples. The content is structured into modules, providing a comprehensive overview of the subject matter.

Uploaded by

Prahlad Sathish
Copyright
© © All Rights Reserved
We take content rights seriously. If you suspect this is your content, claim it here.
Available Formats
Download as PDF, TXT or read online on Scribd
0% found this document useful (0 votes)
277 views56 pages

Laplace_Transform

The document is an educational resource on Laplace Transforms, authored by Jeeja A. V. for a mathematics course at the University of Kerala. It covers various topics including the definition, existence, linearity, and transforms of elementary functions, along with proofs and examples. The content is structured into modules, providing a comprehensive overview of the subject matter.

Uploaded by

Prahlad Sathish
Copyright
© © All Rights Reserved
We take content rights seriously. If you suspect this is your content, claim it here.
Available Formats
Download as PDF, TXT or read online on Scribd
You are on page 1/ 56

Laplace Transforms

Jeeja A. V.
Assistant Professor
Department of Mathematics
Govt. K.N.M Arts and Science College, Kanjiramkulam

E-Resource of Mathematics

FDP in MATHEMATICS under Choice Based Credit and Semester System,


University of Kerala
According to the syllabus for 2018 Admission
Semester - VI
MM 1645: Integral Transforms - Module I
2

Contents
1 Laplace transform 3

2 Laplace transform of some elementary functions 4

3 Transforms of Derivatives and Integrals 15

4 Unit Step Function (Heaviside Function) 26

5 Unit impulse function 31

6 Periodic Functions 32

7 Convolution 34

8 Differentiation and Integration of Transforms 39

9 Inverse Laplace Transforms 47

References 56

Jeeja A. V. [email protected]
3

1 LAPLACE TRANSFORM
Rb
A relation of the form F(s) = a k(s,t) f (t) dt which transforms a given function f (t) into
another function F(s), is called an integral transform. Here K(s,t) is called the kernal of the
transform and F(s) the transform of f (t). The most common integral transforms are

(i) a = 0, b = ∞, K(s,t) = e−st , (Laplaces)


1
(ii) a = −∞, b = ∞, K(s,t) = √ eist (Fourier)

The idea behind any transform is that given problem can be solved more easily in the trans-
formed domain. Laplace transform reduces the problem of solving a differential equation to an
algebraic problem. It is widely used in problems where the

Definition 1.1
Let f (t) be a function defined for all t ≥ 0. The Laplace transform of f (t) is defined as
Z ∞
L [ f (t)] = e−st f (t) dt = F(s), (1)
0

provided the integral exists. Here s is a parameter real or complex. The function f (t)
whose transform is F(s) is said to be the inverse transform of F(s) and is denoted by
L −1 [F(s)]. Thus if L [ f (t)] = F(s), then f (t) = L −1 [F(s)].

EXISTENCE OF LAPLACE TRANSFORM

A function f (t) is said to be of exponential order or satisfies growth restriction if there exist
constants M and a such that | f (t)| ≤ M eat for all positive t.

Theorem 1.1 (Existence Theorem for Laplace Transforms)


If f (t) is defined and piecewise continuous on every finite interval on the semi-axis t ≥ 0
and is of exponential order for all for all t ≥ 0 and some constants M and a, then the
Laplace transform of f (t) exists for all s > a.

Proof. Since f (t) is piecewise continuous, e−st f (t) is integrable over any finite interval on the

Jeeja A. V. [email protected]
4

t-axis. Also f (t) is of exponential order so that for s > a we get,


Z ∞
|L [ f (t)]| = e−st f (t) dt
0
Z ∞
≤ |e−st f (t)| dt
Z0∞
= e−st | f (t)| dt
Z0 ∞
≤ e−st Meat dt
0
Z k
= M lim e−(s−a)t dt
k→∞ 0
" #k
e−(s−a)t
= M lim
k→∞ −(s − a)
0
" #
e−(s−a)k 1
= M lim −
k→∞ −(s − a) −(s − a)
M
= .
s−a
Hence L [ f (t)] exists for s > a.

I Note that Z ∞ Z k
f (x) dx = lim f (x) dx
a k→∞ a

provided, the limit on the right side exist.

LINEARITY OF THE LAPLACE TRANSFORM

The Laplace transform is a linear operation. i.e. for any function f (t) and g(t) whose Laplace
transform exist and any constants a and b,

L [a f (t) + bg(t)] = aL [ f (t)] + bL [g(t)].

Proof. By definition,
Z ∞
L [a f (t) + bg(t)] = e−st [a f (t) + bg(t)] dt
0Z
∞ Z ∞
−st
=a e f (t) dt + b e−st g(t) dt
0 0

= aL [ f (t)] + bL [g(t)].

2 LAPLACE TRANSFORM OF SOME ELEMENTARY


FUNCTIONS

Jeeja A. V. [email protected]
5

I Note that x→∞


lim e−x = 0 and lim e−ax = 0 for any a > 0.
x→∞

1
(i) L (1) = , s > 0.
s

Proof. By definition,
Z ∞
L (1) = e−st 1 dt
0
Z k
= lim e−st 1 dt
k→∞ 0
 −st k
e
= lim
k→∞ −s 0
 −sk 
e 1
= lim +
k→∞ −s s
 
1
= 0+
s
1
= , s > 0.
s

1
(ii) L (eat ) = , s > a.
s−a

Proof.
Z ∞
L [e ] =
at
e−st eat dt
Z0 ∞
= e−(s−a)t dt
0
" #k
e−(s−a)t
= lim
k→∞ −(s − a)
0
" #
e−(s−a)k 1
= lim +
k→∞ −(s − a) s−a
1
= if s > a.
s−a

n!
(iii) L (t n ) = where n is a positive integer.
sn+1
1
Proof. We prove this formula by induction. For n = 0, t n = t 0 = 1 and L (1) = . Thus
s
the result is true for n = 0. We now make the induction hypothesis that it holds for any
n!
positive integer n. i.e. L (t n ) = n+1 .
s

Jeeja A. V. [email protected]
6

Now
Z ∞
L (t n+1
)= e−st t n+1 dt
Z0∞
= t n+1 e−st dt
0
Z k
= lim t n+1 e−st dt
k→∞ 0
" #
e −st k Z k
e−st
= lim t n+1 − (n + 1)t n dt
k→∞ −s 0 0 −s
 −sk Z k −st 
n+1 e ne
= lim k − 0 − (n + 1)t dt
k→∞ −s 0 −s
n + 1 ∞ −st n
Z
= [0 − 0] + e t dt
s 0
n+1
= L (t n )
s
n + 1 n!
=
s sn+1
(n + 1)!
= n+2 .
s

Thus the result is true for n + 1. This proves formula (iii).


a
(iv) L [sin at] = , s>0
s2 + a2
Z ∞ Z k
−st
Proof. We have L [sin at] = e sin at dt = lim e−st sin at dt. We first evaluate
0 k→∞ 0
R −st
e sin at dt. Let I = e−st sin at dt, then
R

− cos at − cos at
Z
I = e−st − (−s)e−st dt
a a
−e−st cos at s
Z
= − e−st cos at dt
a a
−e−st cos at

s −st sin at −st sin at
Z
= − e − (−s)e dt
a a a a
−e−st cos at s −st s2
Z
= − 2 e sin at − 2 e−st sin at dt
a a a
−e−st cos at s −st s2
= − 2 e sin at − 2 I
a a a
s2 −e−st cos at s −st
⇒I+ I= − 2 e sin at
a2 a a
(s2 + a2 )I −st
−e cos at s
⇒ = − 2 e−st sin at
a2 a a
⇒ (s2 + a2 )I = −ae−st cos at − se−st sin at

= e−st (−a cos at − s sin at)


e−st
⇒I= (−a cos at − s sin at).
s2 + a2

Jeeja A. V. [email protected]
7

Hence,
Z k
L [sin at] = lim e−st sin at dt
k→∞ 0
k
e−st

= lim 2 (−s sin at − a cos at)
k→∞ s + a2 0
 −sk 
e 1
= lim 2 (−s sin ak − a cos ak) − 2 (0 − a)
k→∞ s + a2 s + a2
a
= 0+ 2
s + a2
a
= 2 , s > 0.
s + a2

I We can see that for s > 0,


lim e−sx sin ax = 0
I x→∞
lim e−sx cos ax = 0
I x→∞

Proof. We have

sin ax 1
e−sx sin ax = sx
≤ sx = e−sx .
e e

Since e−sx → 0 as x → ∞, we see that lim e−sx sin ax = 0.


x→∞

Similarly lim e−sx cos ax = 0, since | cos ax| ≤ 1.


x→∞

s
(v) L (cos at) = , s > 0.
s2 + a2
Z k
Proof. L [cos at] = lim e−st cos at dt. As in the previous problem, we can see that
k→∞ 0
R −st e−st
e cos at dt = s2 +a2
(−s cos at + a sin at). Using it,
Z ∞
L [cos at] = e−st cos at dt
0
 −st k
e
= lim 2 (−s cos at + a sin at)
k→∞ s + a2 0
 −sk 
e 1
= lim 2 (−s cos ak + a sin ak) − 2 (−s + 0)
k→∞ s + a2 s + a2
s
= 0+ 2
s + a2
s
= 2 .
s + a2

a
(vi) L [sinh at] = , s > |a|
s2 − a2

Jeeja A. V. [email protected]
8

eat − e−at
Proof. We have sinh at =
2
 
1 +at −at
L [sinh at] = L (e − e )
2
1
= {L (eat ) − L (e−at )}
2 
1 1 1
= −
2 s − a s − (−a)
 
1 s + a − (s − a)
=
2 (s − a)(s + a)
1 2a
= × 2
2 s − a2
a
= 2 .
s − a2

s
(vii) L [cosh at] = , s > |a|
s2 − a2

Proof.

eat + e−at
 
L [cosh at] = L
2
1
= {L (eat ) + L (e−at )}
2 
1 1 1
= +
2 s−a s+a
 
1 s+a+s−a
=
2 s2 − a2
1 2s
= × 2
2 s − a2
s
= 2 .
s − a2

Problem 2.1

 et , 0 < t < 1
Find the Laplace transform of f (t) =
 0 when t > 1

Jeeja A. V. [email protected]
9

Solution. We have
Z ∞
L { f (t)} = e−st f (t) dt
0
Z 1 Z ∞
−st
= e f (t) dt + e−st f (t) dt
0 1
Z 1 Z ∞
= e−st et dt + e−st 0 dt
0 1
Z 1
= e−(s−1)t dt
0
 −(s−1)t 1
e
=
−(s − 1) 0
e−(s−1) 1
= −
−(s − 1) −(s − 1)
e1−s 1
= −
1−s 1−s
e1−s − 1
= .
1−s

Problem 2.2

 cost, 0 < t < 2π
Find the Laplace transform of f (t) =
 0, t > 2π

Solution. We have
Z ∞
L { f (t)} = e−st f (t) dt
0
Z 2π Z ∞
−st
= e f (t) dt + e−st f (t) dt
0 2π
Z 2π Z ∞
= e−st cost dt + e−st 0. dt
0 2π
2π
e−st

= (−s cost + sint)
s2 + 1 0
e−s2π 1
= 2 (−s) − 2 (−s)
s +1 s +1
s
= (1 − e−2sπ ) 2 .
s +1

Problem 2.3
Find the Laplace transform of the following functions.
(i) sin 3t cos 2t (iii) cos3 2t (v) sin 3t sin 2t
(ii) sin2 3t (iv) sinh3 2t (vi) e−4t − 6t 2 + 4 sin 2t

Jeeja A. V. [email protected]
10

Solution. 1. We have
1
L [sin 3t cos 2t] = {L (sin 5t) + L (sint)}
2 
1 5 1
= +
2 s2 + 25 s2 + 1
3(s2 + 5)
= 2
(s + 1)(s2 + 2s)
2. we have
 
1 − cos 6t 1
L (sin 3t) = L
2
= {L (1) − L (cos 6t)}
2 2
 
1 1 s 18
= − 2 = 2
2 s s + 36 s(s + 36)
1 3
3. we have cos 3A = 4 cos3 A − 3 cos A or cos3 A = cos 3A + cos A
4 4
1 3
∴ cos3 2t = cos 6t + cos 2t
4 4
1 3
L (cos3 2t) = L (cos 6t) + L (cos 2t)
4 4
1 s 3 s
= × 2 + × 2
4 s + 36 4 s + 4
s(s2 + 28)
=
(s2 + 4)(s2 + 36)

4. we have sinh 3A = 3 sinh A + 4 sinh3 A.


1 3
i.e. sinh3 A = sinh 3A − sinh A
4 4
1 3
i.e. sinh3 2t = sinh 6t − sinh 2t
4 4
1 3
∴ L [sinh3 2t] = L [sinh 6t] − L[sinh 2t]
4  4 
1 6 3 2
= −
4 s2 − 36 4 s2 − 4
3 s2 − 4 − s2 + 36
= × 2
2 (s − 36)(s2 − 4)
48
= .
(s − 4)(s2 − 36)
2

1
5. We have sin A sin B = [cos(A − B) − cos(A + B)]
2
1
i.e. sin 3t sin 2t = [cost − cos 5t]
2
1
∴ L [sin 3t sin 2t] = [L (cost) − L [(cos 5t)]
2 
1 s s
= −
2 s2 + 1 s2 + 25
12s
= 2
(s + 1)(s2 + 25)

Jeeja A. V. [email protected]
11

6. We have

L [e−4t − 6t 2 + 4 sin 2t] = L (e−4t ) − 6L (t 2 ) + 4L (sin 2t)


1 2! 2
= − 6 × 2+1 + 4 × 2
s − (−4) s s +4
1 12 8
= − 3+ 2 .
s+4 s s +4

Problem 2.4
Find the Laplace transforms of

1. 3e5t + (t + 2)2 + 2 cos 3t

2. sin(at + b)

Solution. 1. L [3e5t + (t + 2)2 + 2 cos 3t] = 3L (e5t ) + L [(t + 2)2 ] + 2L [cos 3t] (1)

1
But L [e5t ] =
s−5
L ((t + 2)2 ) = L [t 2 + 4t + 4] = L (t 2 ) + 4L (t) + L (4)
2! 1! 1 2 4 4
= 3
+4· 2 +4× = 3 + 2 + .
s s s s s s
s
and L [cos 3t) = 2
s +9

Substituting in (1)

3 2 4 4 2s
L [3e5t + (t + 2)2 + 2 cos 3t] = + 3+ 2+ + 2
s−5 s s s s +9

2. sin(at + b) = sin at cos b + cos at sin b

∴ L [sin(at + b)] = cos bL [sin at] + sin bL (cos at)


a s
= cos b × 2 2
+ sin b × 2
s +a s + a2
a cos b + s sin b
=
s2 + a2

EXERCISE

Find the Laplace transform of the following.

Jeeja A. V. [email protected]
12


 t/k, 0 < t < k
(1) f (t) =
 1, t > k

 sint, 0 < t < π
(2) f (t) =
 0, t > π

(3) cost · cos 2t

(4) sin 2t cos 3t

(5) 5e3t + 3t 3 − 2 sin 3t + 3 cos 3t

(6) sin3 2t

(7) cos(at + b)

(8) cosh3 2t

(9) cos2 3t + (t 2 + 1)2

ANSWERS
1 − e−ks 1 + e−πs s(s2 + 5)
(1) (2) (3)
ks2 s2 + 1 (s2 + 1)(s2 + 9)
2(s2 − 5) 5 18 3(s − 2)
(4) (5) + 4+ 2
(s2 + 1)(s2 + 25) s−3 s s +9
48 s cos b − a sin b
(6) (7)
(s2 + 4)(s2 + 36) s2 + a2
s(s2 − 28) 2
s + 18 24 4 1
(8) (9) + 5 + 3+
(s2 − 4)(s2 − 36) 2
s(s + 36) s s s

Theorem 2.1 (First shifting theorem, s-Shifting)


If f (t) has the transform F(s) (where s > k for some k), then eat f (t) has the transform
F(s − a) (where s − a > k). In formulas,

L [eat f (t)] = F(s − a)

or, if we take the inverse on both sides,

eat f (t) = L −1 [F(s − a)]

Jeeja A. V. [email protected]
13

Proof. We have
Z ∞
F(s) = L [ f (t)] = e−st f (t) dt (1)
Z0 ∞
∴ L [eat f (t)] = e−st eat f (t) dt
Z0∞
= e−(s−a)t f (t) dt
0

= F(s − a), by (1).

I Note that L [e−at f (t)] = F(s + a).


Problem 2.5
Find the Laplace transform of the following:

(i) e−3t t 3

(ii) e−2t cos2 t

(iii) sinh at · sin at

(iv) e−2t [cos 4t + 3 sin 4t]

(v) et cosh 3t

(vi) 5e2t sinh 2t

(vii) (t + 1)2 et

3! 6
Solution. (i) We have L [t 3 ] = 4
= 4 . By shifting property, we get
s s
6
L [e−3t t 3 ] = (Replacing s by s + 3)
(s + 3)4

1 + cos 2t
(ii) We have cos2 t =
2
 
1 1 1 s
∴ L [cos t] = L [1 + cos 2t] =
2
+ .
2 2 s s2 + 4

By shifting property,
 
−2t 1 1 s+2
L [e 2
cos t] = + (Replacing s by s + 2)
2 s + 2 (s + 2)2 + 4

Jeeja A. V. [email protected]
14

eat − e−at
(iii) We have sinh at = so that
2
eat − e−at
 
sinh at · sin at = sin at
2
1
= [eat sin at − e−at sin at]
2
1
∴ L [sinh at sin at] = L [eat sin at] − L [e−at sin at]} (1)
2
a a
But L [sin at] = 2 2
. By shifting property, L [eat sin at] = and L [e−at sin at] =
s +a (s − a)2 + a2
a
.
(s + a)2 + a2
Substituting in (1),
 
1 a a
L [sinh at sin at] = −
2 (s − a)2 + a2 (s + a)2 + a2

s 4
(iv) We have L [cos 4t + 3 sin 4t] = +3×
s2 + 16 s2 + 16
12 + s
= .
s2 + 16

By shifting property,

12 + (s + 2) s + 14
L [e−2t (cos 4t + 3 sin 4t)] = =
(s + 2)2 + 16 s2 + 4s + 20

e3t + e−3t e4t + e−2t


 
t t
(v) we have e cosh 3t = e =
2 2
1
∴ L [et cosh at] = [L (e4t ) + L (e−2t )]
2  
1 1 1
= × +
2 s−4 s+2
s−2
= 2
s − 2s − 8

e2t − e−2t
 
5
(vi) 5e2t sinh 2t = 5e2t = [e4t − 1]
2 2

 
5 5 1 1
∴ L [5e sinh 2t] = [L (e ) − L (1)] =
2t 4t

2 2 s−4 s
10
=
s(s − 4)

or
2 10
We have L [5 sinh 2t] = 5 × = .
s2 − 4 s2 − 4

Jeeja A. V. [email protected]
15

By shifting property,

10 10
L [e2t 5 sinh 2t] = =
(s − 2)2 − 4 s2 − 4s + 4 − 4
10
=
s(s − 4)

(vii) L [(t + 1)2 ] = L (t 2 + 2t + 1) = L (t 2 ) + 2L (t) + L (1)

2! 1 1 2 + 2s + s2
= + 2 × + =
s2 s2 s s3

2 + 2(s − 1) + (s − 1)2
By shifting property, L [et (t + 1)2 ] =
(s − 1)3
s2 + 1
= .
(s − 1)3

Problem 2.6
1 s
If L [ f (t)] = F(s), then show that L [ f (at)] = F
a a

Proof.
Z ∞
L [ f (at)] = e−st f (at) dt put at = u a dt = du
0
du
Z ∞
= e−su/a · f (u) ·
0 a
1
= F(s/a).
a

3 TRANSFORMS OF DERIVATIVES AND INTE-


GRALS
The Laplace transform is a method of solving ODEs and initial value problems. The cru-
cial idea is that operations of calculus on functions are replaced by operations of algebra on
transforms. Roughly, differentiation of f (t) will correspond to multiplication of L ( f ) by s
and integration of f (t) to division of L ( f ) by s. To solve ODEs, we must first consider the
Laplace transform of derivatives. You have encountered such an idea in your study of loga-
rithms. Under the application of the natural logarithm, a product of numbers becomes a sum of
their logarithms, a division of numbers becomes their difference of logarithms.

Jeeja A. V. [email protected]
16

Theorem 3.1
If L [ f (t)] = F(s), then the transforms of the first and second derivatives of f (t) satisfy

L [ f 0 (t)] = sF(s) − f (0)

L [ f 00 (t)] = s2 F(s) − s f (0) − f 0 (0).

The first equation holds if f (t) is continuous for all t ≥ 0 and satisfies the growth re-
striction and f 0 (t) is piecewise continuous on every finite interval on the semi-axis t ≥ 0.
Similarly, second equation holds if f and f 0 are continuous for all t ≥ 0 and satisfy the
growth restriction and f 00 is piecewise continuous on every finite interval on the semi-axis
t ≥ 0.

Proof. By definition,
Z ∞
L [ f 0 (t)] = e−st f 0 (t)dt
0

Integrating by parts

0
 −st k Z ∞ −st
L [ f (t)] = lim e f (t) 0 − e (−s) f (t)dt
k→∞ 0
h i Z∞
−sk
= lim e f (k) − f (0) − e−st (−s) f (t)dt
k→∞ 0
Z ∞
= 0 − f (0) + s e−st f (t)dt
0

= − f (0) + s · L [ f (t)].

Thus
L [ f 0 (t)] = sL [ f (t)] − f (0).

Now using the formula for L [ f 0 (t)], we get

L [ f 00 (t)] = − f 0 (0) + sL [ f 0 (t)]

= − f 0 (0) + s [− f (0) + s · L [ f (t)]]

= − f 0 (0) − s f (0) + s2 L [ f (t)].

Hence
L [ f 00 (t)] = s2 F(s) − s f (0) − f 0 (0).

I By applying the above result to higher order derivatives we get,


L [ f 000 ] = s3 L [ f ] − s2 f (0) − s f 0 (0) − f 00 (0)

Jeeja A. V. [email protected]
17

and so on. In general,

Theorem 3.2
Let f , f 0 , . . . , f (n−1) be continuous for all t ≥ 0 and satisfy the growth restriction. Fur-
thermore, let f be piecewise continuous on every finite interval on the semi-axis t ≥ 0.
Then the transform of f (n) satisfies

L [ f (n) ] = sn L [ f ] − sn−1 f (0) − sn−2 f 0 (0) − · · · − s f (n−2) (0) − f (n−1) (0). (2)

TRANSFORMS OF INTEGRALS
Theorem 3.3
Let F(s) denote the transform of a function f (t) which is piecewise continuous for t ≥ 0
and satisfies a growth restriction. Then, f ors > 0, s > k, and t > 0,
Z t  Z t  
F(s) −1 F(s)
L f (u)du = , f (u)du = L (3)
0 s 0 s

Proof. By definition,
Z t  Z ∞
Z t

−st
L f (u)du = e f (u)du dt
0 0 0
Z ∞ Z t

= f (u)du e−st dt
0 0
Z k Z t 
= lim f (u)du e−st dt
k→∞ 0 0
" Z  −st k Z k #
t e e−st
= lim f (u)du − f (t) dt
k→∞ 0 −s 0 0 −s
 −sk Z 0
e−st
Z k  Z k 
e 1
= lim f (u)du − f (u)du − f (t) dt
k→∞ 0 −s 0 −s 0 −s
0 1 e−st
Z ∞
= L { f (t)} − (0) − f (t) dt
−s −s 0 −s
1 ∞ −st
Z
= (0 − 0) + e f (t)dt
s 0
F(s)
= .
s

Problem 3.1
1
If L [ f (t)] = , find f (t)
s(s2 + w2 )

Jeeja A. V. [email protected]
18

Solution. Let
1
F(s) =
s2 + w2
Then
F(s) 1
= 2
s s(s + w2 )
 
−1 1
∴ f (t) = L
s(s2 + w2 )
  Zt
−1 F(s)
=L = L −1 [F(s)]dt
s 0
Z t  
−1 1
= L dt
0 s2 + w2
1 − cos wt t
Z t  
sin wt
= dt =
0 w w w 0
1 1 − cos wt
= − 2 [cos wt − 1] = .
w w2
Problem 3.2
Find  
−1 1
L
s2 (s2 + w2 )

Solution. Let
1
F(s) =
s2 + w2
Then  
−1 −1 1 sin wt
L (F(s)) = L =
s + w2
2 w
Now
   Zt
−1 1 F(s)−1
L = L = L −1 [F(s)]ds
s(s2 + w2 ) s 0
1 − cos wt t
Z t  
sin wt
= dt =
0 w w w 0
−1 1 − cos wt
= (cos wt − 1) =
w w2

1 G(s) 1
Let G(s) = , so = 2 2 .
s(s2 + w2 ) s s (s + w2 )
    Zt
−1 1 −1 G(s)
∴L = L = L −1 [G(s)]dt
s2 (s2 + w2 ) s 0
sin wt t
Z t  
1 − cos wt 1
= dt = 2 t −
0 w2 w w 0
 
1 sin wt
= 2
t− .
w w

Jeeja A. V. [email protected]
19

EXERCISE

Find the inverse transform of the following  


1 1 9 s+1
(i) (ii) (iii)
s2 + 4s s3 − s s2 s2 + 9
1 s2 + 3 1
(iv) 2
(v) (vi)
s(s + 1) s(s2 + 9) s(s + a)
s2 + 2 1 1
(vii) (viii) (ix)
s(s2 + 4) s2 (s + 1) s(s2 − 16)

Answers
1
(i) (1 − e−4t ) (ii) cosht − 1
4
1
(iii) 1 + t − cos 3t − sin 3t (iv) 1 − cost
3
1 1 − e−at
(v) [1 + 2 cos 3t] (vi)
3 a
(vii) cos2 t (viii) t − 1 + e−t
1
(ix) [cosh 4t − 1]
16

DIFFERENTIAL EQUATIONS, INITIAL VALUE PROBLEMS

Let us now discuss how the Laplace transform method solves ODEs and initial value problems.
We consider an initial value problem

y00 + ay0 + by = r(t), y(0) = K0 , y0 (0) = K1 (4)

where a and b are constants. Here r(t) is the given input (driving force) applied to the mechani-
cal or electrical system and y(t) is the output (response to the input) to be obtained. In Laplace’s
method we do three steps:

Step 1: Apply Laplace Transform to (4) and obtain

[s2Y − sy(0) − y0 (0)] + a[sY − y(0)] + bY = R(s)

where Y = L (y), R(s) = L (r). From this we get the subsidiary equation

(s2 + as + b)Y = (s + a)y(0) + y0 (0) + R(s).

Jeeja A. V. [email protected]
20

Step 2: Using completing square method, rewrite

a2
 
2
 a 2
s + as + b = s + + b−
2 4

and so we get
(s + a)y(0) + y0 (0) + R(s)
Y= 2  2

s + a2 + b − a4

Step 3: Reduce RHS of the previous equation (usually by partial fractions as in calculus) to a
sum of terms whose inverses can be found, so that we obtain the solution y(t) = L −1 (Y ).

The method will be clear from the following examples.


Problem 3.3
d2 y dy
Using Laplace transforms, find the solution of the initial value problem + 2 + 5y =
dt 2 dt
e−t sint, given y(0) = 0, y0 (0) = 1.

Solution. Given equation is y00 + 2y0 + 5y = e−t sint. Taking Laplace transforms of both sides,

L (y00 ) + 2L (y0 ) + 5L (y) = L (e−t sint). (1)

But L (y0 ) = sL (y) − y(0) and L (y00 ) = s2 L (y) − sy(0) − y0 (0).


Since y(0) = 0 and y0 (0) = 1, L (y0 ) = sL (y) and L (y00 ) = s2 L (y) − 1.

1
Also L (sint) =
s2 + 1
1
So L [e−t sint] = (by shifting theorem)
(s + 1)2 + 1
1
= 2
s + 2s + 2

Substituting the above values in (1) we get

1
[s2 L (y) − 1] + 2[sL (y)] + 5L (y) =
s2 + 2s + 2
1
i.e. (s2 + 2s + 5)L (y) = +1
s2 + 2s + 2
1 + s2 + 2s + 2
=
s2 + 2s + 2
s2 + 2s + 3
= 2
s + 2s + 2
(s2 + 2s + 3)
∴ L (y) =
(s2 + 2s + 2)(s2 + 2s + 5)

Jeeja A. V. [email protected]
21

On resolving the RHS into partial fractions, we get

1/3 2/3
L (y) = +
s2 + 2s + 2 s2 + 2s + 5
1/3 2/3
= +
(s + 1) − 1 + 2 (s + 1)2 − 1 + 5
2

1/3 2/3
= +
(s + 1) + 1 (s + 1)2 + 4
2
   
1 −1 1 2 −1 1
∴ y= L + L
3 (s + 1)2 + 1 3 (s + 1)2 + 22
1 −t 2 sin 2t
= e · sint + × e−t
3 3 2
1 −t
= [e sint + e−t sin 2t].
3

Problem 3.4
Using Laplace transforms, solve y00 + 3y0 + 2y = 8 cos 2t, given that y(0) = −1, y0 (0) = 2.

Solution. Given equation is y00 + 3y0 + 2y = 8 cos 2t. Taking Laplace transforms of both sides

L (y00 ) + 3L (y0 ) + 2L (y) = 8L (cos 2t)


s
[s2 L (y) − sy(0) − y0 (0)] + 3[sL (y) − y(0)] + 2L (y) = 8 2
s +4
8s
(s2 L (y) + s − 2 + 3sLy) + 3 + 2L (y) = 2
s +4
∴ y(0) = −1, y0 (0) = 2

8s
L (y)[s2 + 3s + 2] = −s−1
s2 + 4
8s − s3 − 4s − s2 − 4
i.e. L (y)(s2 + 3s + 2) =
s2 + 4
−s3 − s2 + 4s − 4
i.e. L (y) = 2 (1)
(s + 4)(s2 + 3s + 2)

Let

−s3 − s2 + 4s − 4 −s3 + −s2 4s − 4


=
(s2 + 4)(s2 + 3s + 2) (s2 + 4)(s + 1)(s + 2)
As + B C D
= 2 + +
s +4 s+1 s+2

Multiplying both sides by (s2 + 4)(s + 1)(s + 2), we get

−s3 − s2 + 4s − 4 = (As + B)(s + 1)(s + 2) +C(s2 + 4)(s + 2) + D(s2 + 4)(s + 1).

Jeeja A. V. [email protected]
22

Put s = −1,

1 − 1 − 4 − 4 = c(1 + 4)(−1 + 2)

−8 = 5c

⇒ c = −8/5

Put s = −2,

+8 − 4 − 8 − 4 = D(4 + 4)(−2 + 1)

−8 = −8D

⇒D = 1

Equating the coefficients of s3 ,


8 −2
−1 = A +C + D ⇒ A = −1 −C − D− = −1 + − 1 =
5 5
Equating the constant terms,

−4 = 2B + 8C + 4D
64 24
⇒ 2B = −4 − 8C − 4D = −4 + −4 =
5 5
12
⇒B=
5
−2 12
s+
∴ L (y) = 5 2 5 + −8/5 + 1
s +4 s+1 s+2
−2 s 12 1 8 1 1
= 2
+ 2
− +
5 s + 4 5 s + 4 5 s +  1 s + 2    
2 −1 s 12 −1 1 8 −1 1 −1 1
∴y=− L + L − L +L
5 s2 + 4 5 s2 + 4 5 s+1 s+2
2 12 sin 2t 8 −t
= − cos 2t + − e + e−2t .
5 5 2 5
Problem 3.5
d2 y dy
Using Laplace transform method solve 2
+ 2 + 6y = 6te−t , given that y(0) = 2,
dt dt
y0 (0) = 5.

Solution. Given equation is y00 + 2y0 + 6y = 6te−t . Taking Laplace transform of both sides,

L (y00 ) + 2L (y0 ) + 6L (y) =6L (e−t t)


1
[s2 L (y) − sy(0) − y0 (0)] + 2[sL (y) − y(0)] + 6L (y) = 6
(s + 1)2
6
s2 L (y) − 2s − 5 + 2sL (y) − 4 + 6L (y) =
(s + 1)2

Jeeja A. V. [email protected]
23

6
(s2 + 2s + 6)L (y) = + 2s + 9
(s + 1)2
2s3 + 13s2 + 20s + 15
=
(s + 1)2
2s3 + 13s2 + 20s + 15
∴ L (y) = (1)
(s + 1)2 (s2 + 2s + 6)

2s3 + 13s2 + 20s + 15 A B Cs + D


Let 2 2
= + 2
+ 2 . Multiplying both sides by (s +
(s + 1) (s + 2s + 6) s + 1 (s + 1) s + 2s + 6
1)2 (s2 + 2s + 6), we get

2s3 + 13s2 + 20s + 15 = A(s + 1)(s2 + 2s + 6) + B(s2 + 2s + 6) + (cs + D)(s + 1)2 .

Put s = −1,

−2 + 13 − 20 + 15 = B(1 − 2 + 6)

6 = 5B ⇒ B = 6/5

Equating the coefficients of s3


2 = A +C (2)

Equating the coefficients of s2


13 = 3A + B + 2C + D (3)

Equating the coefficients of s,


20 = 8A + 2B +C + 2D (4)

Equating the coefficients terms


15 = 6A + 6B + D (5)

Solving (2), (3) and (4), (5) we get

A = 0, C = 2, D = 39/5
2s3 + 13s2 + 20s + 15 6/5 2s + 39/5
Therefore = +
(s + 1)2 (s2 + 2s + 6) (s + 1)2 s2 + 2s + 6
6/5 2(s + 1) − 2 + 39/5
= 2
+
(s + 1) (s + 1)2 + 5
6/5 (s + 1) 29/5
= 2
+2 +
(s + 1) (s + 1) + 5 (s + 1)2 + 5
2

Jeeja A. V. [email protected]
24

Substituting in (1),
6 1 s+1 29 1
L (y) = · +2 2
+
5 (s + 1)2 (s + 1) + 5 5 (s + 1)2 + 5
     
6 −1 1 −1 s+1 29 −1 1
or y = L + 2L + L
5 (s + 1)2 (s + 1)2 + 5 5 (s + 1)2 + 5

6 √ 29 sin 5t
= te−t + 2e−t cos 5t + × e−t √
5 5 5
−t √ √
 
e 29
= 6t + 10 cos 5t + √ sin 5t .
5 5

Problem 3.6
d2 y dy
Solve the differntial equation by using Laplace transform − 3 + 2y = 4, given
dt 2 dt
y(0) = 2, y0 (0) = 3.

Solution. Given equation is y00 − 3y0 + 2y = 4. Taking Laplace transforms of both sides,

L (y00 ) − 3L (y0 ) + 2L (y) = L (4)

{s2 L (y) − sy(0) − y0 (0)} − 3[sL (y) − y(0)] + 2L (y) = 4L (1)


4
s2 L (y) − 2s − 3 − 3sL (y) + 6 + 2L (y) =
s

4 2s2 − 3s + 4
L (y)[s2 − 3s + 2] = + 2s − 3 =
s s
2
2s − 3s + 4 2s2 − 3s + 4
L (y) = 2 = (1)
s(s − 3s + 2) s(s − 1)(s − 2)
2
2s − 3s + 4 A B C
Let = + +
s(s − 1)(s − 2) s s − 1 s − 2
2.02 − 3.0 + 4 4
where A = = =2
(0 − 1)(0 − 2) 2
2.12 − 3.1 + 4
B= = −3
1(1 − 2)
2.22 − 3 × 2 + 4 8 − 6 + 4
C= = =3
2(2 − 1) 2×1
2s2 − 3s + 4 2 −3 3
∴ = + +
s(s − 1)(s − 2) s s − 1 s − 2
Substituting in (1)
2 3 3
L (y) = − +
s s −1  s − 2    
−1 1 −1 1 −1 1
∴ y = 2L − 3L + 3L
s s−1 s−2
= 2 − 3et + 3e2t .

Jeeja A. V. [email protected]
25

EXERCISE

Solve the following differential equations by using Laplace transform.

(1) y00 − 3y0 + 2y = 4t + e3t when y(0) = 1, y0 (0) = −1

(2) x00 − 2x0 + x = et , where x(0) = 2, x0 (0) = −1


d2 y dy
(3) + 4 + 3y = e−t , y(0) = y0 (0) = 1
dt 2 dt
(4) y00 + 2y0 − 3y = sint, given y(0) = y0 (0) = 0
d3 y d2 y dy dy d2 y
(5) + 2 − 2 − 2y = 0 when y = 1, = 2, and = 2 at t = 0.
dt 3 dt 2 dt dt dt 2
(6) y00 + 2y0 + y = te−t if y(0) = 1, y0 (0) = −2
d3 y d2 y dy
(7) + 2 − − 2y = 0, if y(0) = y0 (0) = 0 and y00 (0) = 6.
dx3 dx2 dx
(8) y00 + 2y0 + y = e−t if y(0) = 0 and y0 (0) = 1

(9) y000 − y0 = 2 cost, y(0) = 3, y0 (0) = 2 and y00 (0) = 1


d2 y 12
(10) 2
+ 9y = cos 2t, y(0) = 1 and y0 (0) =
dt 5
(11) 2y00 + 5y0 + 2y = e−2t , y(0) = y0 (0) = 1

(12) y00 + 2y0 + 2y = 5 sint, y(0) = y0 (0) = 0

ANSWERS
1
(1) y = 3 + 2t + (e3t − et ) − 2e2t
2
1
(2) x = 2et − 3tet + t 2 et
2
1
(3) y = (7e−t − 3e−3t − 2te−t )
4
1 1 1
(4) y = et − e−3t − (2 sint + cost)
8 40 10
5 1
(5) y = et − e−t + e−2t
3 3
(6) y = e−t (1 − t + t 3 /6)

(7) y = ex − 3e−x + 2e−2x

Jeeja A. V. [email protected]
26

e−t 2
(8) y = (t + 2t)
2
(9) y = 2 + 2et − e−t − sint
1 4 4
(10) y = cos 2t + cos 3t + sin 3t
5 5 5
20 −t/2 11 −2t 1 −2t
(11) y = e − e − te
9 9 3
(12) y = 2e−t cost + e−t sint − 2 cost + sint

4 UNIT STEP FUNCTION (HEAVISIDE FUNCTION)

The unit step function u(t − a) is defined as follows:



 0 when t < a
u(t − a) = , a ≥ 0.
 1 when t ≥ a

The unit step function is also called the Heaviside function.

Laplace transform of unit step function


e−as
L [u(t − a)] = .
s
Proof.
Z t
By definition, L [u(t − a)] = e−st u(t − a)dt
0
Z a Z ∞
= e−st 0dt + e−st 1dt
0 a
Z k
= 0 + lim e−st dt
k→∞ a
 −st k
e
= lim
k→∞ −s a
e −sa
= 0−
−s
e−as
= .
s

Jeeja A. V. [email protected]
27

Problem 4.1
Express the following function in terms of unit step function and find its Laplace trans-
form 
 8, t < 3
f (t) =
 5, t > 3

Solution.

 8 + 0, t < 3
f (t) =
 8 − 3, t > 3

 0, t < 3
= 8+
 −3, t > 3

 0, t < 3
= 8 + (−3)
 1, t > 3

= 8 − 3u(t − 3)

∴ L [ f (t)] = L (8 − 3u(t − 3)]

= 8L (1) − 3L(u(t − 3))


8 e−3s
= −3·
s s

Theorem 4.1 (Second shifting theorem; t shifting)


If f (t) has the Laplace
 transform F(s), then the shifted function
 0 if t < a
˜f (t) = f (t − a)u(t − a) = has the transform e−as F(s).
 f (t − a) if t > a

Proof.

L [ f˜(t)] = L [ f (t − a)u(t − a)]


Z ∞
= e−st f (t − a) · u(t − a)dt
0
Z a Z ∞
−st
= e 0̇dt + e−st f (t − a)dt
Z0 ∞
a

= e−st f (t − a)dt
0

Jeeja A. V. [email protected]
28

Put t − a = u, so that dt = du. When t = a, u = 0 and t → ∞ ⇒ u → ∞.


Z ∞
∴ L [ f˜(t)] = e−s(u+a) f (u)du
0 Z ∞
−as
=e e−su f (u)du
0

= e−as F(s).

I Since L [ f (t − a)u(t − a)] = e−asF(s), L −1[e−asF(s)] = f (t − a)u(t − a).


I Any piecewise continuous function



 f0 (t) if 0 < t < t1


f1 (t) if t1 < t < t2




..

f (t) = .





 fn−1 (t) if tn−1 < t < tn



 f (t) if t < t < ∞.
n n

defined on 0 < t < ∞ can be given by the single expression

f (t) = f0 (t)[u(t − 0) − u(t − t1 )] + f1 (t)[u(t − t1 ) − u(t − t2 )] + · · ·

+ fn−1 (t)[u(t − tn−1 ) − u(t − tn )] + fn (t) u(t − tn ).

I Effects of the unit step function



 0 if t < a,
u(t − a) =
 1 if t > a.

 0 if t < a,
f (t)u(t − a) =
 f (t) if t > a.

and

 0 if t < a,
f (t − a)u(t − a) =
 f (t − a) if t > a.

Problem 4.2
Express the following function in terms of unit step function and hence find its Laplace
transform 


 2 if 0<t <π

f (t) = 0 if π < t < 2π



 sint if t > 2π.

Jeeja A. V. [email protected]
29

Solution. The given function f (t) can be expressed as

f (t) = 2[u(t − 0) − u(t − π)] + 0[u(t − π) − u(t − 2π)] + sint · u(t − 2π)

= 2u(t) − 2u(t − π) + sint · u(t − 2π)

= 2u(t) − 2u(t − π) + sin(t − 2π) · u(t − 2π)


2 2e−πs e−2πs
∴ L ( f (t)) = − + 2 .
s s s +1

Problem 4.3
Express the following function in terms of unit step function



 2 + t 2 if 0 < t < 2

f (t) = 6 if 2 < t < 3
2


if t > 3.


2t − 5

Solution. The given function f (t) can be expressed as

2
f (t) = (2 + t 2 )[u(t − 0) − u(t − 2)] + 6[u(t − 2) − u(t − 3)] + · u(t − 3)
  2t − 5
32 − 12t
= (2 + t 2 )u(t) + (4 − t 2 ) u(t − 2) + · u(t − 3).
2t − 5

Problem 4.4
2 2e−2s 4e−2s se−πs
Find the inverse transform of F(s) = − 2 − + 2 .
s2 s s s +1

     
1 1 s
Solution. We have L −1 = t, L −1 = 1 and L −1 = cost. Now
s2 s s2 + 1

L −1 [e−as F(s)] = f (t − a) · u(t − a)

where f (t) = L −1 (F(s)). This shows that,

L −1 [e−as F(s)] = f (t − a) · u(t − 2),


 
−1 −2s 1
L e = 1 · u(t − 2),
s
 
−1 −πs s
L e = cos(t − π)u(t − π)
s2 + 1
 
−1 −2s 1
L e = (t − 2)u(t − 2).
s2

Jeeja A. V. [email protected]
30

Thus
2 2e−2s 4e−2s se−πs
 
−1 −1
L [F(s)] = L − 2 − + 2
s2 s s s +1
   −2s   −2s   −πs 
−1 1 −1 e −1 e −1 se
= 2L − 2L − 4L +L
s2 s2 s s2 + 1
= 2t − 2(t − 2)u(t − 2) − 4u(t − 2) + cos(t − π)u(t − π)

= 2t − 2tu(t − 2) − costu(t − π)



 2t if 0 < t < 2

= 0, if 2 < t < π


 − cost if t > π

Problem 4.5
Find the inverse Laplace transform of the following:
e−πs
(i)
s2 + 1
se−s/2 + πe−s
(ii)
s2 + π 2
3 4e−s 4e−3s
(iii) − 2 + 2
s s s

 
1
Solution. (i) We have L −1 2
= sint. Since L −1 [e−as F(s)] = f (t − a)u(t − a)
s +1
where f (t) = L −1 [F(s)],
 
−1 1
L e−πs 2
= sin(t − π) · u(t − π)
s +1
= − sint · u(t − π)

1
s− 1
se 2 + πe−s − s s π
(ii) 2 2
=e 2 2 2
+ e−s 2 . (1)
s +π s +π s + π2
   
s π
Now L −1 = cos πt and L −1 = sin πt.
s2 + π 2 s2 + π 2
By second shifting property,
1
 
− s
   
−1  s  1 1
L e 2 2 = cos π t − ·u t −
s + π2 2 2
 
1
= sin πt · u t −
2
 
π
and L −1 e−s · 2 = sin π(t − 1)u(t − 1)
s + π2
= − sin πt · u(t − 1).

Jeeja A. V. [email protected]
31

 1 
s

 se 2 + πe−s 
By (1), L −1 
 s2 + π 2 

 
1
= sin πt · u t − − sin πt · u(t − 1)
2
   
1
= sin πt u t − − u(t − 1)
2
3 1 1
(iii) Let F(s) = − 4e−s · 2 + 4e−3s 2 . (1)
 s s  s
1 1
Now L −1 = 1, L −1 = t.
s s2
By second shifting properties,
 
−1 −s 1
L e 2 = (t − 1)u(t − 1) and
s
 
−1 −3s 1
L e = (t − 3)u(t − 3).
s2
By (1), L −1 [F(s)] = 3 − 4(t − 1)u(t − 1) + 4(t − 3)u(t − 3).

5 UNIT IMPULSE FUNCTION

The unit impulse function is considered as the limiting form of the function of

 1/k if a ≤ t ≤ a + k
δk (t − a) = (5)
 0 otherwise.

This function is represented in the figure. In mechanics, the impuse of a force f (t) over a
time interval, a ≤ t ≤ a + k is defined to be the integral of f (t) from a to a + k. Thus the impulse
Ik of the function (1) is
Z a+k
Ik = δk (t − a)dt
a
Z a+k
1
= dt
a k
1
= (t)a+k
k a
1
= × k = 1.
k

Jeeja A. V. [email protected]
32

The limit of δk (t − a) as k → 0 (k > 0) is denoted by δ (t − a) and it is called the unit impuse


function or Dirac delta function. Thus the unit impulse function δ (t − a) is defined as follows.

 ∞ if t = a Z ∞
δ (t − a) = and δ (t − a)dt = 1.
 0 if t 6= a. 0

By the definition, the Laplace transform of δ (t − a) can be obtained as


Z ∞
L [δ (t − a)] = e−st δ (t − a)dt
0 Z

= lim e−st δk (t − a)dt
k→0 0
Z a+k
1
= lim e−st dt
k→0 a k
 −st a+k
1 e
= lim
k→0 k −s a
e−s(a+k) − e−as
= lim
k→0 −ks
e (1 − e−ks ) e−as
−as 1 − e−sk
= lim = lim
k→0 ks s k→0 k
e −as −sk
e s
= lim (by L’Hospitals rule)
s k→0 1
= e−as .

When a = 0, L [δ (t)] = e0 = 1.

6 PERIODIC FUNCTIONS

If f (t) is a periodic function with period T , then


Z T
1
L [ f (t)] = e−st f (t)dt fors > 0. (6)
1 − e−sT 0

Proof. By definition,
Z ∞
L [ f (t)] = e−st f (t)dt
0
Z T Z 2T Z 3T
−st −st
= e f (t)dt + e f (t)dt + e−st f (t)dt + · · · .
0 T 2T

Substituting t = u + T in second integral and t = u + 2T in third integral and so on.


Z T Z T Z T
−st −s(u+T )
L [ f (t)] = e f (t)dt + e f (u + T )du + e−s(u+2T ) f (u + 2T )du + · · ·
0 0 0
Z T Z T Z T
= e−su f (u)du + e−sT e−su f (u)du + e−2sT e−su f (u)du + · · ·
0 0 0
Z T
= (1 + e−sT + e−2sT + · · · ) e−su f (u)du
0
Z T  
1 −st 1
= e f (t)dt ∵ 1 + x + x2 + · · · = .
1 − e−sT 0 1−x

Jeeja A. V. [email protected]
33

Problem 6.1
Find the Laplace transform of the saw-toothed wave function of period T , defined by
kt
f (t) = , 0 < t < T .
T

Solution. We have

1
Z T
L [ f (t) = e−st f (t)dt
1 − e−sT 0
Z T
1 kt
= −sT
e−st dt
1−e 0 T
Z T
k
= −sT
te−st dt
T (1 − e ) 0
" T Z T −st #
k e−st e
= −sT
t − 1 dt
T (1 − e ) −s 0 0 −s
"  −st T #
k Te −sT 1 e
= −sT
+
T (1 − e ) −s s −s 0
 −sT 
k Te 1 −sT
= − 2 (e − 1) .
T (1 − e−sT ) −s s

Problem 6.2
Find the Laplace transform of the triangular wave function of period 2a given by

 t, 0 < t < a
f (t) =
 2a − t, a < t < 2a

Solution.

1
Z T
L ( f (t)] = e−st f (t)dt
1 − e−sT 0
Z 2a
1
= e−st f (t)dt
1 − e−s·2a 0
Z a Z 2a 
1 −st −st
= e tdt + e (2a − t)dt
1 − e−2as 0 a
(  −st a   −st 2a )
1 e−st e e−st e
= −2as
t· −1 2
− (2a − t) − (−1)
1−e −s s 0 −s s2 a
1 as
= tanh .
s2 2

Jeeja A. V. [email protected]
34

Problem 6.3
Find the Laplace transform of the Half-wave rectifier function

 sin wt f or 0 < t < π/w
f (t) = 2π
 0 f or π/w < t <
w


Solution. f (t) is a periodic function with period T = .
w
1
ZT
Let L [ f (t)] = e−st f (t)dt
1 − e−sT 0
Z π/w Z 2π/w 
1 −st −st
= e sin wtdt + e odt
2π 0 π/w
−s( )
1−e w
 −st π/w
1 e
= (−s sin wt − w cos wt)
2π s2 + w2 0
−s( )
1−e w ( )
1 e−sπ/w · ω w
= + 2 .
−2πs s2 + w2 s + w2
1−e w

7 CONVOLUTION

The convolution of f and g is denoted by f ∗ g and is defined as


Z t
f ∗g = f (u)g(t − u)du. (7)
0

Also f ∗ g = g ∗ f .

Theorem 7.1 (Convolution Theorem)


If two functions f and g satisfy the assumption in the existence theorem, so that their
transforms F and G exist, the product H = FG is the transform of h = f ∗ g. From this
Z t
−1
L (F(s)G(s)) = f ∗ g = f (u)g(t − u)du.
0

Jeeja A. V. [email protected]
35

Problem 7.1
Apply convolution theorem to evaluate the inverse Laplace transform of the following:
s
(i)
(s2 + a2 )2

s2
(ii)
(s2 + a2 )(s2 + b2 )
1
(iii)
(s2 + a2 )2

s2
(iv)
(s2 + 4)2
1
(v)
s(s2 − a2 )

s s 1 s
Solution. (1) Let = × = f1 (s) f2 (s) where f1 (s) = and
(s2 + a2 )2 s2 + a2 s2 + a2 s2 + a2
1
f2 (s) = .
s2 + a2
Now

f (t) = L −1 [ f1 (s)]
 
−1 s
=L = cos at
s2 + a2

and

g(t) = L −1 [ f2 (s)]
1
= L −1 [ ]
s2 + a2
sin at
= .
a
By convolution theorem,
 
−1 s
L 2 2 2
= L −1 [ f1 (s) f2 (s)]
(s + a )
Z t
= f (t − u)g(u)du
0
Z t
sin au
= cos a(t − u) du
0 a
1 t
Z
= [sin at + sin(2au − at)]du
2a 0
 t
1 1
= u sin at − cos(2au − at)
2a 2a 0
1
= t sin at.
2a

Jeeja A. V. [email protected]
36

s s
(2) Let f1 (s) = and f 2 (s) =
s2 + a2 s2 + b2
 
−1 s −1
f (t) = L ( f1 (s)) = L = cos at
s2 + a2
 
−1 −1 s
g(t) = L ( f2 (s)) = L = cos bt
s2 + b2

s2
 
−1
∴L
(s2 + a2 )(s2 + b2 )
= L −1 [ f1 (s) f2 (s)]
Z t
= f (t − u)g(u)du, by convolution theorem
0
Z t
= cos a(t − u) cos budu
0
1 t
Z
= [cos(at − (a − b)u) + cos(at − (a + b)u)]du
2 0
1 sin(at − (a − b)u) sin(at − (a + b)u) t
 
= +
2 −(a − b) −(a + b) 0
 
1 sin bt − sin at (sin at + sin bt)
=− −
2 a−b a+b
a sin at − b sin bt
=
a2 − b2

1 1
(3) Let f1 (s) = and f 2 (s) = .
s2 + a2 s2 + a2
Then

f (t) = L −1 [ f1 (s)]
 
−1 1
=L
s2 + a2
sin at
=
a
g(t) = L −1 [ f2 (s)]
 
−1 1
=L
(s2 + a2 )
sin at
= .
a

Jeeja A. V. [email protected]
37

By convolution theorem, we have


 
−1 1
L 2 2
= L −1 [ f1 (s) f2 (s)]
(s + a )
Z t
= f (t − u)g(u)du
0
Z t
sin a(t − u) sin au
= · du
0 a a
1 t
Z
= sin(at − au) · sin audu
a2 0
Z t
1
= [cos(at − 2au) − cos(at)]du
2a2 0
 t
1 sin(at − 2au)
= − u cos at
2a2 −2a u=0
 
1 2 sin at
= × − t cos at
2a2 2a
1
= [sin at − at cos at]
2a3
s s
(4) Let f1 (s) = and f2 (s) = .
s2 + 4 s2 + 4
 
s
Then f (t) = L −1 [ f 1 (s)] = L −1 = cos 2t
s2 + 22
and g(t) = L −1 [ f2 (s)] = cos 2t

By convolution theorem, we have

s2
 
−1
L = L −1 [ f1 (s) · f2 (s)]
(s2 + 4)2
Z t
= f (t − u) · g(u)du
0
Z t
= cos(2t − 2u) · cos 2udu
0
1 t
Z
= [cos 2t + cos(2t − 4u)]du
2 0
sin(2t − 4u) t
 
1
= u cos 2t +
2 −4 u=0
 
1 sin 2t
= t cos 2t +
2 2

1 1
(5) Let f1 (s) = and f2 (s) = 2 .
s s − a2
 
1
Then f (t) = L [ f1 (s)] = L
−1 −1 = 1 and
 s
1 sinh at
g(t) = L −1 [ f2 (s)] = L −1 2 = .
s − a2 a

Jeeja A. V. [email protected]
38

By convolution theorem, we have


 
−1 1
L 2 2
= L −1 [ f1 (s), f2 (s)]
s(s − a )
Z t
= f (t − u) · g(u)du
0
Z t  t
sinh au 1 cosh au
= ·1 · du =
0 a a a 0
1
= 2 (cosh at − 1)
a

EXERCISE

(i) Apply convolution theorem to find the inverse Laplace transform of

1 1 3
(i) (ii) (iii)
(s + a)(s + b) s(s2 + 4) (s2 + 1)(s2 + 9)
s s2 2
(iv) (v) (vi)
(s2 + 1)(s2 + 4) s4 − a4 (s + 1)(s2 + 4)
1 1 1
(vii) (viii) (ix)
(s + 1)(s + 9)2 (s − 2)(s − 3) (s + 1)(s2 + 1)
1
(x)
s (s + a2 )
2 2

(ii) Find the Laplace tranform of the following

(i) t 2 u(t − 2) (ii) sint · u(t − 4)

(iii) Find the inverse Laplace transforms of

se−as e−2s
(i) (ii)
s2 − w2 s−3
e−s se−2s e−s
(iii) (iv) (v)
(s − 1)(s − 2) s2 − 1 (s + 1)3

Find the Lapalce transform of the square-wave function of period a defined as f (t) =
(iv) 
 1 for 0 < t < a/2
.
 −1 for a/2 < t < a

2t
(v) Find the Laplace transform of the wave form f (t) = , 0 ≤ t ≤ 3.
3

Jeeja A. V. [email protected]
39

(vi) Find the Laplace transform of the periodic function f (t) = et for 0 < t < 2π.

(vii) Find the Laplace transform of



 cos wt, for 0 < t < π/w
f (t) =
 0 for π/w < t < 2π/w

ANSWERS

e−bt − e−at 1
1. (i) (ii) (1 − cos 2t)
a−b 4
1 1
(iii) [3 sint − sin 3t] (iv) (cost − cos 2t)
8 3
1 2e−t 1
(v) (sinh at + sin at) (vi) − (2 cos 2t − sin 2t)
2a 5 5
e−t
(vii) [1 − e−8t (1 + 8t)] (viii) e2t − e3t
64
1 −t 1
(ix) [e + sint − cost] (x) (at − sin at)
2 a3
e−2s 2 e−4s
2. (i) (4s + 4s + 2) (ii) (cos 4 + s sin 4)
s3 s2 + 1
3. (i) cosh w(t − a) · u(t − a) (ii) e3(t−2) u(t − 2)

(iii) [e2(t−1) − et−1 ]u(t − 1) (iv) cosh(t − 2) · u(t − 2)

1 −(t−1)
(v) e (t − 1)2 u(t − 1)
2
1  as 
4. tanh
s 4
2e−3s 2
5. +
−s(1 − e−3s ) 3s2
e2(1−s)π − 1
6.
(1 − s)(1 − e−2sπ )
s
7.
(s2 + w2 )(1 − e−πs/w )

8 DIFFERENTIATION AND INTEGRATION OF


TRANSFORMS

Jeeja A. V. [email protected]
40

Theorem 8.1 (Differentiation of Transforms/Multiplication by t n )


dn
If L [ f (t)] = F(s), then L [t n f (t)] = (−1)n [F(s)] where n = 1, 2, 3, . . . .
dsn

Proof. Given
Z ∞
F(s) = L [ f (t) = e−st · f (t) dt (1)
0

Differentiating (1) w.r.t ‘s’, we get


Z 
d d ∞
−st
[F(s)] = e · f (t) dt
ds ds 0
Z ∞

= [e−st ] f (t) dt
Z0∞
∂s
= −t e−st · f (t) dt
0Z

=− e−st [t f (t)] dt
0

= −L [t f (t)].
d
or L [t f (t)] = (−1)1 [F(s)] (2)
ds
d2
Similarly, L [t 2 f (t)] = (−1)2 2 [F(s)]
ds
d3
L [t 3 f (t)] = (−1)3 3 [F(s)].
ds
..
.
dn
In general, L [t n f (t)] = (−1)n [F(s)].
dsn

Problem 8.1
Find the Laplace transform of the following

(i) te−t cost

(ii) t 2 sin wt

(iii) t sinh at

(iv) t 2 et · sin 4t

(v) te2t cos 5t

(vi) t sin2 3t

Jeeja A. V. [email protected]
41

Solution. (i) We have


s
L [cost] =
s2 + 1  
d s
∴ L [t cost] = (−1)
ds s2 + 1
[(s2 + 1) · 1 − s · 2s] s2 − 1
= (−1) =
(s2 + 1)2 (s2 + 1)2
By shifting property, we get

L [e−t t cost] = {L [t cost]}s→s+1


(s + 1)2 − 1 s2 + 2s
= =
[(s + 1)2 + 1]2 (s2 + 2s + 2)2
w
(ii) We have L [sin wt] =
s2 + w2
d2
 
w
∴ L [t sin wt] = (−1) 2 2
2 2
ds s + w2
  
d d w
=
ds ds s2 + w2
 
d −2ws
=
ds (s2 + w2 )2
 2
(s + w2 )2 × 1 − s × 2(s2 + w2 ) × 2s

= −2w
(s2 + w2 )4
 2
s + w2 − 4s2
= −2w
(s2 + w2 )3
6ws2 − 2w3
= 2
(s + w2 )3
a
(iii) we have L [sinh at] =
s2 − a2
 
d a 2as
∴ L [t sinh at] = (−1) 2 2
= 2
ds s − a (s − a2 )2

4
(iv) we have L [sin 4t] =
s2 + 16
d2
 
4
∴ L [t sin 4t] = (−1) 2 2
2 2
ds s + 16
  
d d 1
= 4×
ds ds s2 + 16
 
d −2s
= 4×
ds (s2 + 16)2
 2
(s + 16)2 1 − s × 2(s2 + 16) × 2s
= −8
(s2 + 16)4
24s2 − 128
= 2
(s + 16)3

Jeeja A. V. [email protected]
42

By shifting property, we get

L [et · t 2 sin 4t] = {L [t 2 sin 4t]}s→s−1


24(s − 1)2 − 128
=
((s − 1)2 + 16)3
24s2 − 48s − 104
=
(s2 − 2s + 17)3

s
(v) we have L [cos 5t] =
s2 + 25
 
d s
so L [t cos 5t] = (−1)
ds s2 + 25
 2
s2 − 25

(s + 25) − s × 2s
= (−1) =
(s2 + 25)2 (s2 + 25)2

By shifting property, we get

L [e2t t cos 5t] = {L [t cos 5t]}s→s−2


(s − 2)2 − 25
=
((s − 2)2 + 25)2
s2 − 4s − 21
= 2
(s − 4s + 29)2

1 − cos 6t
(vi) we have sin2 3t =
2
So
 
1 1 1 s
L [sin 3t] = [L (1) − L (cos 6t)] =
2

2 2 s s2 + 36
  
d 1 1 s
∴ L [t sin 3t] = (−1)
2

ds 2 s s2 + 36
  2 
1 1 (s + 36)1 − s × 2s
=− − 2−
2 s (s2 + 36)2
36 − s2
 
1 1
= 2+ ×
2s 2 (s2 + 36)2

Theorem 8.2 (Integration of Transforms /Division by t)


  Z∞
f (t)
If L [ f (t)] = F(s), then L = F(s) ds.
t s

Jeeja A. V. [email protected]
43

Proof. Given F(s) = L [ f (t)] =


R ∞ −st
0 e f (t) dt. Integrating both sides w.r.t. s between s and ∞.
Z ∞ Z ∞ Z ∞ 
−st
F(s) ds = f (t) dt ds.
e
s s 0
Z ∞ Z ∞ 
−st
= e f (t) ds dt
0 s
e−st ∞
Z ∞ 
= f (t) dt
0 −t s
e−st
Z ∞ 
= 0 − f (t) dt
0 −t
 
−st f (t)
Z ∞
= e dt
0 t
 
f (t)
=L .
t

Hence the result

EXAMPLE 8

Find the Laplace transform of the following.


1 − et e−at − e−bt
(i) (ii)
t t
sint cos at − cos bt
(iii) (iv)
t t
1 − cost
(v)
t2

Solution
1 1
(i) We have L [1 − et ] = L (1) − L (et ) = −
s s−1
1 − et
  Z∞
∴ L = L (1 − et ) ds
t s
Z ∞ 
1 1
= − ds = {log s − log(s − 1)}∞
s
s s s−1
  ∞   ∞
s 1
= log = log
s−1 s
1
1− s
  s
1
= log 1 − log
1
1−
 s  
s s−1
= 0 − log = log
s−1 s

Jeeja A. V. [email protected]
44

1 1
(ii) we have L [e−at − e−bt ] = −
s+a s+b
 −at
e − e−bt
 Z∞
∴ L = L [e−at − e−bt ] ds
t s
Z ∞ 
1 1
= − ds
s s+a s+b
= {log(s + a) − log(s + b)}∞
s
∞
s+a
= log
s+b s
1 + a/s ∞
 
= log
1 + b/s s
 
1 + a/s
= log 1 − log
1 + b/s
 
s+a
= 0 − log
s+b
 
s+b
= log
s+a

1
(iii) we have L [sint] =
s2 + 1
  Z∞
sint
∴ L = L (sint) ds
t s
1
Z ∞
= 2
ds
s s +1
π
= tan−1 (s)|∞
s = − tan−1 s
2
π
= − tan−1 s
2
= cot−1 (s) = tan−1 (1/s)

Jeeja A. V. [email protected]
45

s s
(iv) we have L [cos at − cos bt] = −
s2 + a2 s2 + b2
  Z∞
cos at − cos bt
∴ L = L [cos at − cos bt] ds
t s
Z ∞ 
s s
= − ds
s s2 + a2 s2 + b2
 ∞
1 2 2 1 2 2
= log(s + a ) − log(s + b )
2 2 s
 2 2 ∞
1 s +a
= log 2
2 s + b2 s
1 1 + a2 /s2 ∞
= log
2 1 + b2 /s2 s
1 + a2 /s2
  
1
= log 1 − log
2 1 + b2 /s2
 2
s + a2
 
1
= 0 − log 2
2 s + b2
1 s2 + b2
= log 2
2 s + a2

1 s
(v) we have L [1 − cost] = − 2
s s +1
  Z∞
1 − cost
∴ L = L[1 − cost] ds
t s
Z ∞ 
1 s
= − ds
s s s2 + 1
 ∞
1 2
= log s − log(s + 1)
2 s
1
= [log s2 − log(s2 + 1)]∞ s
2 
s2 ∞

1
= log 2
2 s +1 s
 ∞   
1 1 1 1
= log = 0 − log
2 1 + 1/s2 s 2 1 + 1/s2
  2   2 
1 s 1 s
= − log 2 = − log 2
2 s +1 2 s +1

Jeeja A. V. [email protected]
46

Again

  1 − cost  
1 − cost t
L =L
t2 t
Z ∞  
1 − cost
= L ds
s t
 2 
1 s
Z ∞
= − log 2 ds
s 2 s +1
 2 
1 ∞ s
Z
=− log 2 1 ds
2 s s +1

Integrating by parts
  2  ∞ Z ∞
1 s 1
=− log 2 s −
2 s +1 s s s2 
s2 + 1
[(s2 + 1)2s − s2 · 2s]

× s ds
(s2 + 1)2
∞ Z ∞ 2
2s2
  
1 1 s +1
=− s log − × 2 ds
2 1 s s s2 (s + 1)2
1+ 2
 s  Z ∞ 
1 1 2
=− 0 − s · log − ds
2 1 2
s s +1
1+ 2
  2 s 
1 s −1 ∞
= − −s log 2 − 2(tan (s))s
2 s +1
 2 
s s π
= log 2 + − tan−1 s
2 s +1 2
 2 
s s π
= log 2 + − tan−1 s
2 s +1 2
 2 
s s
= log 2 + cot−1 s
2 s +1

EXERCISE

Find the Laplace transform of the following

(1) cosh at sin at (2) e−t cos2 t


(3) t 3 e2t (4) e−t (2 cos 5t − 3 sin 5t)
1
(5) 2 × e−t cos2 t (6) e4t sin 2t cost
2
(7) cosh 3t cos 2t (8) e−t (sin 2t − 2t cos 2t)
(9) e−t cos2 t sin 3t (10) t 2 e−2t
Jeeja A. V. [email protected]
47

sin2 t sinht
(11) (12)
t t
e−t sint
(13) t e−2t cost
2 (14)
t
eat − cos bt
(15)
t
cos 2t − cos 3t
(16) (17) t 2 e−3t sin 2t
t

ANSWERS

a(s2 + 2a2 ) 1 s+1


(1) (2) + 2
s4 + 4a4 2s + 2 2s + 4s + 10
6 2s − 13
(3) (4) 2
(s − 2)4 s + 2s + 26
 
1 1 3 1
(5) 2
(6) +
(s + 1)(s + 2s + 2) 2 (s − 4)2 + 9 (s − 4)2 + 1
2s(s2 − 5) 16
(7) 4 2
(8)
s − 10s + 169 (s + 2s + 5)2
2
 
1 6 5 1
(9) + +
4 (s + 1)2 + 9 (s + 1)2 + 25 (s + 1)2 + 1
2 1 s2 + 4
(10) (11) log
(s + 2)4 4 s2
2(s3 + 10s2 + 25s + 22)
 
1 s−1
(12) log (13)
2 s+1 (s2 + 4s + 5)3
 2
s − b2

−1 1
(14) cot (s + 1) (15) log
2 s−a
 2
2(s3 + 6s2 + 9s + 2

1 s +9
(16) log 2 (17)
2 s +4 (s2 + 4s + 5)3

9 INVERSE LAPLACE TRANSFORMS

If L [ f (t)] = F(s), then L −1 [F(s)] = f (t), where L− 1 is called the inverse Laplace trans-
form operator. From the application point of view, the inverse Laplace transform is very useful.

Important formulae
 
1
(1) L −1 =1
s
t n−1
 
1
(2) L −1 =
sn (n − 1)!
 
1
(3) L −1 = eat
s−a

Jeeja A. V. [email protected]
48

 
1
(4) L −1 = e−at
s+a
 
1 sin at
(5) L −1 =
s2 + a2 a
 
s
(6) L −1 = cos at
s2 + a2
 
1 sinhat
(7) L −1
2 2
=
s −a a
 
s
(8) L −1 = cosh at
s2 − a2
(9) Shifting property:
L −1 [F(s − a)] = eat L −1 [F(s)]

(10) Multiplicationby t 
d
t f (t) = L −1 − L ( f (t))
ds
(11) Division by t  
f (t)
Z ∞
−1
=L L( f (t))ds
t s

EXAMPLE 9

Find the inverse transform of the follwing

s2 + 2s + 5 2s2 − 6s + 5
(i) (ii)
s3 s3 − 6s2 + 11s − 6
4s + 5 2s + 5
(iii) (iv)
(s + 1)2 (s − 2) 2
s + 4s − 5
2s + 5 5s + 3
(v) 2
(vi)
s + 4s + 13 (s − 1)(s2 + 2s + 5)
s2 + 6 s
(vii) (viii)
(s2 + 1)(s2 + 4) s + 4a4
4
 
s s+1
(ix) (x) log
s + s2 + 1
4 s−1
 
1+s −1 s
 
(xi) log (xii) cot
s 2

Jeeja A. V. [email protected]
49

Solution
s2 + 2s + 5 s2 2s 5 1 2 5
(i) = + + = + +
s3 s3 s3 s3 s s2 s3
 2       
−1 s + 2s + 5 −1 1 −1 1 −1 1
∴L = L + 2L + 5L
s3 s s2 s3
t t2
= 1+2× +5×
1! 2!
t n−1
 
−1 n
∵ L (t ) =
(n − 1)!
5 2
= 1 + 2t + t
2

2s2 − 6s + 5 2s2 − 6s + 5
(ii) Let 3 =
s − 6s2 + 11s − 6 (s − 1)(s − 2)(s − 3)
A B C
= + +
s−1 s−2 s−3
Then (by resolving into partial fraction)

2 · 12 − 6 · 1 + 5
Putting s = 1, A = = 1/2
(1 − 2)(1 − 3)
2 · 22 − 6 · 2 + 5
Putting s = 2, B = = −1
(2 − 1)(2 − 3)
2 · 32 − 6 · 3 + 5
Putting s = 3, c = = 5/2.
(3 − 1)(3 − 2)
2s2 − 6s + 5 1/2 −1 5/2
∴ 3 = + +
s − 6s2 + 11s − 6 s−1 s−2 s−3
2s2 − 6s + 5
     
−1 1 −1 1 −1 1
∴L = L −L
s3 − 6s2 + 11s − 6 2 s−1 s−2
 
5 1
+ L −1
2 s−3
1 t 5
= e − e2t + e3t
2 2

4s + 5 A B C
(iii) Let = + +
(s + 1)2 (s − 2) s + 1 (s + 1)2 s − 2
Multiplying both sides by (s + 1)2 (s − 2),

4s + 5 = A(s + 1)(s − 2) + B(s − 2) + c(s + 1)2

Put s = 2, 13 = 9C ⇒ C = 13/9
1
Put s = −1, 1 = −3B ⇒ B = −
3

Equating the coefficients of s2 ,

0 = A +C ⇒ A = −C = −13/9.

Jeeja A. V. [email protected]
50

4s + 5 −13/9 −1/3 13/9


∴ = + +
(s + 1)2 (s − 2) s+1 (s + 1)2 (s − 2)
     
−1 4s + 5 13 −1 1 1 −1 1
L =− L − L
(s + 1)2 (s − 2) 9 s+1 3 (s + 1)2
 
13 1
+ L −1
9 s−2
13 −t 1 −t 13 2t
= − e − te + e .
9 3 9
2s + 5 2s + 5 A B
(iv) Let = = +
s2 + 4s − 5 (s − 1)(s + 5) s − 1 s + 5
Then
2·1+5
A = = 7/6
1+5
2 · (−5) + 5 −5
B = = = 5/6
(−5 − 1) −6
2s + 5 7/6 5/6
∴ 2 = +
 s + 4s − 5 s − 1 s + 5   
2s + 5 7 −1 1 5 −1 1
i.e. L −1 2 = L + L
s + 4s − 5 6 s−1 6 s+5
7 t 5 −5t
= e + e
6 6
2s + 5
(v) Let F(s) = where the denominator is not factorizable
s2 + 4s + 13
2s + 5 2(s + 2) + 1
F(s) = =
(s + 2)2 − 4 + 13 (s + 2)2 + 9
2(s + 2) 1
= +
(s + 2) + 9 (s + 2)2 + 9
2
   
−1 −1 s+2 −1 1
∴ L [F(s)] = 2L +L
(s + 2)2 + 32 (s + 2)2 + 32
   
−2t −1 s −2t −1 1
= 2e L +e L
s2 + 32 s2 + 32
(∵ L −1 [F(s + a)] = e−at L −1 [F(s)])
sin 3t
= 2e−2t cos 3t + e−2t
3
5s + 3 A Bs +C
(vi) Let 2
= + 2
(s − 1)(s + 2s + 5) s − 1 s + 2s + 5
Multiplying both sides by (s − 1)(s2 + 2s + 5),

5s + 3 = A(s2 + 2s + 5) + (Bs +C)(s − 1)

Put s = 1, 8 = A×8 ⇒ A = 1

Put s = 0, 3 = 5A −C ⇒ C = 5A − 3 = 5 − 3 = 2

Jeeja A. V. [email protected]
51

Equating the coefficients of s2 ,

0 = A + B ⇒ B = −A = −1
5s + 3 1 −s + 2
∴ 2
= + 2
(s − 1)(s + 2s + 5) s − 1 s + 2s + 5
     
−1 5s + 3 −1 1 −1 −s + 2
L = L +L
(s − 1)(s2 + 2s + 5) s−1 s2 + 2s + 5
   
−1 1 −1 −(s + 1) + 3
= L +L
s−1 (s + 1)2 − 1 + 5
   
−1 1 −1 −(s + 1)
= L +L
s−1 (s + 1)2 + 22
 
−1 1
+3L
(s + 1)2 + 22
   
−t −1 s −t −1 1
= e −e L
+t
+ 3e L
s2 + 22 s2 + 22
(∵ L −1 [F(s + a)] = e−t L −1 [F(s)])
sin 2t
= et − e−t cos 2t + 3e−t ·
2

s2 + 6
(vii) Let F(s) = .
(s2 + 1)(s2 + 4)
Since F(s) involves only even powers of s, we put s2 = u
u+6 A B
F(s) = = +
(u + 1)(u + 4) u + 1 u + 4
where
−1 + 6 5 −4 + 6 2
A = = and B= =
−1 + 4 3 −4 + 1 −3
5/3 −2/3 5 1 2 1
∴ F(s) = + = · 2 − · 2
u + 1  u + 4  3 s + 1 3 s + 4
5 −1 1 2 1
∴ L −1 [F(s)] = L 2
− L −1 2
3 s +1 3 s +4
5 sint 2 sin 2t
= · − ·
3 1 3 2
5 1
= sint − sin 2t
3 3

(viii) Since s4 + 4a4 = (s2 )2 + (2a2 )2

= (s2 + 2a2 )2 − 2s2 · 2a2

(∵ A2 + B2 = (A + B)2 − 2AB)

= (s2 + 2a2 )2 − (2as)2

= (s2 + 2a2 + 2as)(s2 + 2a2 − 2as)

Jeeja A. V. [email protected]
52

Let

s s
=
s4 + 4a4 (s2 + 2as + 2a2 )(s2 − 2as + 2a2 )
As + B Cs + D
= 2 2
+ 2
s + 2as + 2a s − 2as + 2a2
−1/4a 1/4a
= 2 2
+ 2
s + 2as + 2a s − 2as + 2a2
1 1 1
=− × 2 2 2
+
4a (s + a) − a + 2a 4a
1
×
(s − a) − a2 + 2a2
2

1 1 1 1
=− × 2 2
+ ×
4a (s + a) + a 4a (s − a)2 + a2
   
−1 s 1 −at −1 1 1
L 4 4
= − ×e ×L 2 2
+
s + 4a 4a s +a 4a
 
1
× eat L −1 2
s + a2
1 sin at 1 sin at
= − e−at + eat
4a a 4a a
sin at at sin at
= (e − e−at ) = × sinh at
4a2 2a2

(ix) Since s4 + s2 + 1 = s4 + 2s2 + 1 − s2

= (s2 + 1)2 − s2

= (s2 + 1 + s)(s2 + 1 − s)

Let
s s As + B Cs + D
= = +
s4 + s2 + 1 (s2 + s + 1)(s2 − s + 1) s2 + s + 1 s2 − s + 1
Multiplying both sides by s4 + s2 + 1,

s = (As + B)(s2 − s + 1) + (Cs + D)(s2 + s + 1)

Equating the coefficients of s3 ,


O = A +C (1)

Equating the coefficients of s2 ,

O = B − A +C + D (2)

Equating the coefficients of s,


1 = A − B +C + D (3)

Jeeja A. V. [email protected]
53

Equating the coefficient terms,


O = B+D (4)

By (4), B + D = O substituting in (2),


(2) becomes
O = −A +C ⇒ A = C

Then (1) becomes,


O = A + A ⇒ 2A = O ⇒ A = O

and hence
C = O.

From (3),
1 = D−B

and by (4),
O = D+B

Adding
1 1
1 = 2D ⇒ D = and B = −D = −
2 2

s −1/2 1/2
∴ = +
s4 + s2 + 1 s2 + s + 1 s2 − s + 1
−1/2 1/2
= +
1 1 1 1
(s + )2 − + 1 (s − )2 − + 1
2 4 2 4
−1/2 1/2
= +
1 2 3 1 3
(s + ) + (s − )2 +
2 4 " 2 4 #
 
s 1 1
∴ L −1 4 2 = − × e−t/2 L −1 √ 2
s +s +1 2 s2 + 3/2
 
1 t/2 −1 1
+ e L √
2 s2 + ( 3/2)2
√ √
1 −t/2 sin 3t/2 1 t/2 sin 3t/2
=− e √ + ×e × √
2 3/2 2 3/2
√ !
2 3 et/2 − e−t/2
= √ sin t
3 2 2

2 3 t
= √ sin t sinh
3 2 2

Jeeja A. V. [email protected]
54

 
s+1
(x) Let L [ f (t)] = F(s) = log = log(s + 1) − log(s − 1)
s−1
Then

d
L [t f (t)] = − L [ f (t)]
ds
d
= − [log(s + 1) − log(s − 1)]
ds
 
1 1
= − −
s+1 s−1
1 1
= −
s−1 s+1 
−1 1 1
∴ t f (t) = L − = et − e−t
s−1 s+1
t
e −e −t  
s+1

et − e−t
−1
∴ f (t) = or L log =
t s−1 t
 
s+1
(xi) Let L [ f (t)] = F(s) = log = log(s + 1) − log s
s
Then   
−1 s+1
f (t) = L log
s

d
But L [t f (t)] = −
L [ f (t)]
ds
d
= − [log(s + 1) − log(s)
ds 
1 1
=− −
s+1 s
1 1
= −
s s+ 1 
−1 1 1
∴ t f (t) =L − = 1 − e−t
s s+1
1 − e−t
∴ f (t) =
t
e−t
  
s + 1 1 −
or L −1 log =
s t

(xii) Let L [ f (t)] = F(s) = cot−1 (s/2)

Then
f (t) = L −1 [cot−1 (s/2)]

Jeeja A. V. [email protected]
55

But

d
L [t f (t)] = − L [ f (t)]
ds
d
= − [cot−1 (s/2)]
ds
 
−2 2
= − 2 2
= 2
s +2 s + 22
 
2
∴ t f (t) = L −1 2 = sin 2t
s + 22
sin 2t
∴ f (t) =
t

EXERCISE 4.3

Find the inverse transform of the following

3(s2 − 1)2 s+1


(i) (ii)
2s5 s2 + s + 1
2s2 − 4 2s2 − 1
(iii) (iv)
(s + 1)(s2 − 5s + 6) (s2 + 1)(s2 + 4)
21s − 33 s3
(v) (vi)
(s + 1)(s − 2)3 s4 − a4
s+8 2s + 1
(vii) 2
(viii)
s + 4s + 5 (s + 2)2 (s − 1)2
 
1 3s − 8
(ix) log 1 + 2 (x) 2
s s − 4s + 20
s+1 s−4
(xi) (xii)
(s + 2s + 2)2
2 4(s − 3)2 + 16
 
1 −1 1
(xiii) (xiv) tan
2(s − 1)2 + 32 s
 2 
s −1
(xv) log (xvi) cot−1 (1 + s)
s2
 2
s + b2

1 1
(xvii) log (xviii)
2 (s − a)2 (s + 1)(s2 + 2s + 2)
s s+3
(xix) (xx)
(s − 1)2
2 (s2 + 6s + 13)2
s2 + s
 
s+a
(xxi) (xxii) log
(s2 + 1)(s2 + 2s + 2) s+b
11s2 − 2s + 5
(xxiii)
2s3 − 3s2 − 3s + 2

ANSWERS

Jeeja A. V. [email protected]
56

1 √ √ !
3 3 2 t4 1 − t √ 3 3
(i) − t + (ii) √ e 2 2 cos t + sin t
2 2 16 3 2 2
7 3t 1 −t 4 2t 3
(iii) e − e − e (iv) sin 2t − sint
2 6 3 2
3 cosh at + cos at
(v) 2e−t − 2e2t + 6te2t + t 2 e2t (vi)
2 2
t(e − e−2t )
t
(vii) e−2t (cost + 6 sint) (viii)
3
2(1 − cost) 1
(ix) (x) e2t (3 cos 4t − sin 4t)
t 2
1 −t 1 3t 1
(xi) te sint (xii) e cos 2t − e3t sin 2t
2 4 8
et sint
(xiii) sin 4t (xiv)
8 t
2 −t
e sint
(xv) (1 − cosht) (xvi)
t t
e−at − cos bt
(xvii) (xviii) e−t (1 − cost)
t
1
(xix) t sinht (xx) 3e−3t t sint
2
1 3
(xxi) (1 + e−t ) sint + (1 − e−t ) cost
5 5
e−bt − e−at 3
(xxii) (xxiii) 2e−t + 5e2t − et/2
t 2

REFERENCES
[1] Erwin Kreyszig, Advanced Engineering Mathematics, 10th Edition, Wiley-India

[2] Peter V. O’ Neil, Advanced Engineering Mathematics, Thompson Publications, 2007

[3] M Greenberg, Advanced Engineering Mathematics, 2nd Edition, Prentice Hall

Jeeja A. V. [email protected]

You might also like